2018-2019 William Carey University College of Osteopathic Medicine (WCUCOM)

This forum made possible through the generous support of SDN members, donors, and sponsors. Thank you.
4th year student here. Feel free to ask questions of me or others about experiences, quality of education and our overall match competitiveness. Best of luck in the up coming cycle and may you select a school that is best for you.

Can you comment on all of the above?? I have a couple of acceptances including WCUCOM, and your thoughts would be helpful in making a decision!
 
Members don't see this ad :)
I'm not able to PM you. Shoot me a message if you're still trying to make a decision. I haven't heard back from the alumnus yet.
You are private as well. I changed my settings though so you should be able to PM now
 
Is anyone deciding between WCUCOM and LMU? any insights into what ya'll think is better. or if anyone is in the same situation, lmk!
 
Can you comment on all of the above?? I have a couple of acceptances including WCUCOM, and your thoughts would be helpful in making a decision!
So I am seeing all these comments on people talking about how the school has changed and doesn't have the problems from the past and most of this is based on their opinions from interviews... This is a serious problem.

1) In earlier threads admins, faculty and deans have all been noted lying to applicants to make the school seem better than it is in regards to things like research, board reviews, curriculum and match rates. The med students apart of the interview crews are the most pro-carey and happiest students. Also it is expected that schools will put their best foot forward and cover up problems. It is wisest to COMPLETELY ignore anything that you learn about a school from interviews that can not be verified over multiple years or from multiple trusted sources.
2) Carey also has a reputation of horrible recruiting methods and when the students don't turn out to be what they wanted they will deliberately try to fail them out. This was going on last year so I have no doubt it is still going on. We still have a very high attrition rate.
3) The biggest problem Carey has going forward is match rates... they are abysmal. The combined match before any post match was under 50%. Many people had to find programs in the SOAP and scramble and a few could not find placement at all. This is a very big deal. I can tell you my class is having problems with interviews, many are just not getting many, those that are doing ok are finding they are all being interviewed by the exact same programs. It is likely that our match rates will be even worse than last year and the rates will continue to decline. It is common that students are pressured to go into fields they don't want when they can not match and last year many did.
This is very important. Each year the match will get harder on DOs and many programs across fields have taken a US MD first dibs approach on interview invites, this is important if you want to do anything other than pathology, family, or peds which often have more spots than they can fill.
The only way to get a bit of an edge is to do rotations with as many programs as you can in your 4th year hoping they like you this is obviously limiting, you can only do so many weeks in one speciality and so much medicine or surgery to met graduation requirements, you only have so many months to get in the auditions, VSAS has limits, you have to be able to get the rotations...ect.
4) Carey doesn't specialize in anything, you get a general osteopathic medical education delivered to you in your 1st two years as lectures, your 3rd year you get preceptor based rotations and your 4th year you are 100% on your own. There is no rural medicine. What is most likely being mistakenly thought of a rural medicine is the possibility of doing rotation in 3rd year in the delta, this is the WORST way to learn as a 3rd year because it lacks the proper structure of a residency program. Its also not going to help you when it comes to getting seen by residency programs especially if you have to make up your education in your 4th year.
5) Prep for boards at this school is non existent. Some claim the lectures have improved which is helpful for step 1 and level 1, maybe it is. I don't know I am not there.


When it comes to picking schools you need 1 thing above everything else SUPPORT from hospital faculty that are a part of residency programs. If you are not in hospitals with residents in your 3rd year your education and connections suffer. 2nd to that schools that teach to the boards and the boards alone are going to help you get stronger board scores. Last you want a school that makes learning easier not harder. Carey still fails all 3 of these things.

I can not speak to ARCOM or most other DO programs, I do know that some have great connections with former Aoa programs and have most of their rotation in those hospitals you can match into and they make it easier to set up. Carey doesn't have that.

The reasons to pick Carey are 1) You have no were else to go 2) You always wanted to be a hattiesburg family doc 3) You want to practice OMM ( Carey is most likely the best place you can go if you want to really use OMM, they will make you very good at it and send you to all kind of conferences ) 4) You want to just get a DO degree and don't care about residency.

.... Thats how I feel about the whole thing.
 
  • Like
Reactions: 2 users
3) The biggest problem Carey has going forward is match rates... they are abysmal. The combined match before any post match was under 50%. Many people had to find programs in the SOAP and scramble and a few could not find placement at all. This is a very big deal. I can tell you my class is having problems with interviews, many are just not getting many, those that are doing ok are finding they are all being interviewed by the exact same programs. It is likely that our match rates will be even worse than last year and the rates will continue to decline. It is common that students are pressured to go into fields they don't want when they can not match and last year many did.
This is very important. Each year the match will get harder on DOs and many programs across fields have taken a US MD first dibs approach on interview invites, this is important if you want to do anything other than pathology, family, or peds which often have more spots than they can fill.
The only way to get a bit of an edge is to do rotations with as many programs as you can in your 4th year hoping they like you this is obviously limiting, you can only do so many weeks in one speciality and so much medicine or surgery to met graduation requirements, you only have so many months to get in the auditions, VSAS has limits, you have to be able to get the rotations...ect.

.... Thats how I feel about the whole thing.

Can you provide any documentation that supports this? I see from their published statistics that SOAP/scramble is lumped into general matching percentage so the amount isn't known.
 
Can you provide any documentation that supports this? I see from their published statistics that SOAP/scramble is lumped into general matching percentage so the amount isn't known.

Only if I absolutely wanted to out myself on the internet which I dont. As you state you can see from there own numbers ( which they used very interesting math) to get 56%. That alone should make you a bit concerned. ( you could say it’s getting better although it isnt because those percentages are about different amounts of people, more people are unmatched with similar class sizes) After the post match and graduation they had every body but 4 or 5 people with a place to go. 2 were offered a 5th year and took it, the others didnt and are currently trying to get matched this year.
Imagine that half of your class did not get into the professional field they were working so hard for and just took a job in something else. Disheartening isnt it?

There were other posts about this on sdn you can dig up if you want to, they may have a few tables that had the school specific match information still up. Sadly that information doesnt stick around too long after the match happens.

Match is going to be hard on DOs at Carey or not, juat gonna be worse at Carey due to some of the things I discussed earlier.

End of the day it is still a med school and if you can make it through they will hand you a valid degree, the other stuff whether they make it harder or not is still on you. Med school is stupid difficult.
 
Only if I absolutely wanted to out myself on the internet which I dont. As you state you can see from there own numbers ( which they used very interesting math) to get 56%. That alone should make you a bit concerned. ( you could say it’s getting better although it isnt because those percentages are about different amounts of people, more people are unmatched with similar class sizes) After the post match and graduation they had every body but 4 or 5 people with a place to go. 2 were offered a 5th year and took it, the others didnt and are currently trying to get matched this year.
Imagine that half of your class did not get into the professional field they were working so hard for and just took a job in something else. Disheartening isnt it?

There were other posts about this on sdn you can dig up if you want to, they may have a few tables that had the school specific match information still up. Sadly that information doesnt stick around too long after the match happens.

Match is going to be hard on DOs at Carey or not, juat gonna be worse at Carey due to some of the things I discussed earlier.

End of the day it is still a med school and if you can make it through they will hand you a valid degree, the other stuff whether they make it harder or not is still on you. Med school is stupid difficult.

I thought the 56% was the people who matched NRMP and 39% AOA, accounting for 95% of the class . That seems different from claiming half the class didn't get into the field they wanted. Am I misreading the numbers or am I missing something else?

Ultimately, as a current student, are you recommending going elsewhere if that's an option?
 
I thought the 56% was the people who matched NRMP and 39% AOA, accounting for 95% of the class . That seems different from claiming half the class didn't get into the field they wanted. Am I misreading the numbers or am I missing something else?

Ultimately, as a current student, are you recommending going elsewhere if that's an option?
If you just took those 2 percentages and added them together then yes you did misread the data. The final total percentage is 99, which is shady because they only counted those that had met graduation requirements and did not include those 2 or 3 others that didnt in the data.
Each match is of those that participated in that match that is how many that matched. The class size was about 96ish students ( cant remember the exact number) 48% matched out right in either match, 98% total placed after post match that were eligible for graduation.
39% are a total pre and post match Aoa while 56% are pre and post match ACGME (the military match is a typo should be 2% and 1% is error from the students last year that did not meet graduation requirements but matched this year), the remaining took 5th year and are currently trying to match and will be included in my data along with those that did not meet graduation requirements.

Am I recommending you go else where if that is an option ( i dont think new aoa schools are an option)? Yes, but that just my opinion based only on this school. If I was you I wouldn’t touch a brand new DO school and would 100% pick Carey if all you have are schools that have yet to graduate at least 4 classes. In this match environment a new aoa school will most likely burn you at Carey you currently have a 50% chance of landing the field you want.
 
  • Like
Reactions: 1 user
If you just took those 2 percentages and added them together then yes you did misread the data. The final total percentage is 99, which is shady because they only counted those that had met graduation requirements and did not include those 2 or 3 others that didnt in the data.
Each match is of those that participated in that match that is how many that matched. The class size was about 96ish students ( cant remember the exact number) 48% matched out right in either match, 98% total placed after post match that were eligible for graduation.
39% are a total pre and post match Aoa while 56% are pre and post match ACGME (the military match is a typo should be 2% and 1% is error from the students last year that did not meet graduation requirements but matched this year), the remaining took 5th year and are currently trying to match and will be included in my data along with those that did not meet graduation requirements.

Am I recommending you go else where if that is an option ( i dont think new aoa schools are an option)? Yes, but that just my opinion based only on this school. If I was you I wouldn’t touch a brand new DO school and would 100% pick Carey if all you have are schools that have yet to graduate at least 4 classes. In this match environment a new aoa school will most likely burn you at Carey you currently have a 50% chance of landing the field you want.

Ok that's good to hear. My only two acceptances right now are WC and a brand new school. So that does make it tricky. Granted the dean of the new school has started a school successfuly before and the administration has been very deliberate and meticulous in their planning but it's still an unknown
 
A current second year here...

I'm going to have to respectfully disagree with WorldChanger's opinion on the school just a little bit (No offense WorldChanger! I have really appreciated your perspective over the years!)

It seems based on my talks with current fourth years and third years that the school changed (matured) quite a bit over the last two years. My experiences have been pretty positive overall. Maybe that's the lifecycle of a school as it ages, but it seems WCUCOM is increasing their professional network as well.

-Network!: We have a ton of very active and involved students in our class; going to conferences, participating in state and national activities, building meaningful relationships with specialty societies, etc. From what I've gathered it seems as maybe earlier classes (as a whole) were not quite as involved in that kind of stuff, but all that network building takes time and as more students graduate out and begin to fill RD positions, or decide to be preceptors, it increases our footprint. I believe we are seeing that happening now. As a student, you have to remember to be active and get involved. Having great boards is not going to cut it anymore, you have to build relationships, you have to get your face out there. There are a lot of opportunities here to get involved and active, go meet people, volunteer, but you have to take the initiative and participate, or you can create those opportunities yourself, which several students in our class have done. Its pretty hard to network if you never leave your house or the library, and you need to do it early. "If you wait until your fourth year then it's too late and you will get lumped in with everyone else. (advice from a friend that is a 2nd-year resident)". If you go pull up a list of the area residency programs, most are at UMMC (teaching hospital in Jackson) but there are like 10 others in various specialties throughout the state and more throughout the area, you will see a lot of Carey grads and Carey grad RDs or fellows. This year they have also added two more full hub-sites which you can do all of your rotations at (one in TN and another in MS, and they may have added a third but I can't remember 100%)

-As for the curriculum, a lot of students are using Pathoma, Sketchy, or Boards and Beyond to supplement or reinforce the classroom material. I can tell you that Boards and Beyond is almost step-for-step in line with what we cover in our basic/clinical science classes. Obviously, the classes go a little deeper into the material than the board prep stuff, but that's one of my resources and I'm doing pretty well in class as well as on my practice questions. It's certainly exhausting keeping up with everything but I feel prepared at this point.

-OPP (What we call OMT or OMM) has done a complete 180 with the new faculty additions. As a whole, they all seem to jell way more and be on the same page than my first year. Last year it seemed disorganized and all over the place. They have made the curriculum and the labs way more clinically / board relevant. I used to dread OPP, but now I like it.

-WCUCOM is not going to hold your hand throughout this process, but at the same time they are not going to try to "Fail you out." I haven't seen that. Unfortunately, there is some attrition, but medical school is tough and you have to hit the ground running. If not you will put yourself in a hole really fast, classes move too quickly. Sometimes the amount of material seems insurmountable, but you will get through it.

-Student Body: I think their selection process works. Our class has talented people from all walks of life, prior life experiences, ages, etc. Some people were probably ready for boards from day one, some people (like me) needed a ton of work. I think it helps make us all more well rounded having to closely work with each other all the time. Each class is around 100 students, which in my mind is just big enough that you don't get tired of seeing each other, but also just small enough that you can get to know everyone if you want. I believe, as a result, we will all become great, knowledgeable, relateable doctors, (minus two or three who are incredibly intelligent, but should probably become pathologists or some other specialty like that ;)).

Also, our associate dean, Dr. Subbarao, was just named the new dean of the school and will take over for Dr. Turner when he retires at the end of this semester. Dr. Subbarao is very well liked and is very receptive to the students and I think this is great for the school.

Overall, I have nothing negative to say about this place. It's not 100% perfect, but then again I don't know of any school that is. I have friends at schools all over the southeast (MD and DO) and each school has minor flaws or things the students wish they could change and we are no different.


It's exhausting and sometimes very humbling, but I really like it here and would 100% recommend it.
 
  • Like
Reactions: 8 users
@WorldChanger36 and @WCUCOM_OMS_Student

I really appreciate you two giving your views and opinions on the school. It really helps to give us prospective or accepted students some perspective when choosing the school. So thank you for taking the time to tell us about your experiences here.
 
  • Like
Reactions: 2 users
Members don't see this ad :)
What are the hub sites. they have a list of 100 hospitals but what are the actual sites..
 
From what I understand, you can make your own hub sites if you get approval
I have heard of people doing this in the past, but from what little I know this is rare. There is a lot of work / red-tape/ that goes into approving a new hub-site and takes a bit of time. If you are coming here or want to know more before applying with regards to hub sites I would reach out to our clinical rotations office.

@doctor2be10


***The following is from our most recent Hub Site information session of which you will go through the selection process during your second year:*****

Required Third Year Curriculum for rotations:

•Family Medicine – 2 months
•Internal Medicine – 2 months
One is general IM, the other can be subspecialty IM if that rotation is available​
•Emergency Medicine – 1 month
•Surgery – 2 months
One is general surgery, the other can be subspecialty surgery if that rotation is available​
•Pediatrics – 1 month
•OB/GYN – 1 month
•Mental Health – 1 month
•* All rotations are based around your hub site location.

Current Third Year Hub-sites:

Mississippi
Columbus*+
Corinth*+
Delta
Gulf Coast
Pine Belt (Hattiesburg/Laurel)+
Capital Region (Jackson)
River Region (Vicksburg)
Keesler**+
McComb
Meridian*+
Tupelo*+

•Alabama
Centreville*+

•Louisiana
West Louisiana+
East Louisiana+


•New York City
New York City (Far Rockaway, NY)*+

•Tennessee
Paris (New Hubsite)

*Indicates that this hub site selects their own student, you MUST apply to these sites.
+Indicates a residency training site

** Military students (notify us if interested)


Fourth Year Rotations: You pick where to go do rotations. just need to make sure your rotations conform to the minimum below requires:
•4th year Elective Rotations
-36 weeks total of clinical experience
-16 weeks in surgery
-16 weeks in medicine
-4 weeks in student choice elective
--Can be medicine, surgery, research, academic or international rotation​
 
Last edited:
  • Like
Reactions: 1 users
I have heard of people doing this in the past, but from what little I know this is rare. There is a lot of work / red-tape/ that goes into approving a new hub-site and takes a bit of time. If you are coming here or want to know more before applying with regards to hub sites I would reach out to our clinical rotations office.

@doctor2be10


***The following is from our most recent Hub Site information session of which you will go through the selection process during your second year:*****

Required Third Year Curriculum for rotations:

•Family Medicine – 2 months
•Internal Medicine – 2 months
One is general IM, the other can be subspecialty IM if that rotation is available​
•Emergency Medicine – 1 month
•Surgery – 2 months
One is general surgery, the other can be subspecialty surgery if that rotation is available​
•Pediatrics – 1 month
•OB/GYN – 1 month
•Mental Health – 1 month
•* All rotations are based around your hub site location.

Current Third Year Hub-sites:

Mississippi
Columbus*+
Corinth*+
Delta
Gulf Coast
Pine Belt (Hattiesburg/Laurel)+
Capital Region (Jackson)
River Region (Vicksburg)
Keesler**+
McComb
Meridian*+
Tupelo*+

•Alabama
Centreville*+

•Louisiana
West Louisiana+
East Louisiana+


•New York City
New York City (Far Rockaway, NY)*+

•Tennessee
Paris (New Hubsite)

*Indicates that this hub site selects their own student, you MUST apply to these sites.
+Indicates a residency training site

** Military students (notify us if interested)


Fourth Year Rotations: You pick where to go do rotations. just need to make sure your rotations conform to the minimum below requires:
•4th year Elective Rotations
-36 weeks total of clinical experience
-16 weeks in surgery
-16 weeks in medicine
-4 weeks in student choice elective
--Can be medicine, surgery, research, academic or international rotation​

All of the hubsites are not complete meaning they require at least one rotation at another site due to a lack of the required rotation. This means expect to travel some to a lot. The newer the site the more you will have to jump from site to site.
Also you can request to do rotations in any site. Use this to your advantage and try to set up intense rotations your 3rd year which will make things easier on you in your 4th year. I recommend trying to find a level 2 trauma for emergency and something not wesley for inpatient IM and OB in jackson. Outside of that it is debatable if other sites may be better to jump to or not.
Also the school is horrible at communicating with preceptors so you will have a few aukward exchanges but be smart and talk to your preceptors before you show up and it will just be an annoyance not and major set back.
 
  • Like
Reactions: 1 users
I have heard of people doing this in the past, but from what little I know this is rare. There is a lot of work / red-tape/ that goes into approving a new hub-site and takes a bit of time. If you are coming here or want to know more before applying with regards to hub sites I would reach out to our clinical rotations office.

@doctor2be10


***The following is from our most recent Hub Site information session of which you will go through the selection process during your second year:*****



So, is this correct? Your 4th year is in a completely different (unrelated to the 3rd year site) location. And you can do electives, all 32 weeks of them at ANY hospital?
-----How does the approval process go there, is it difficult/time consuming? Do you have to contact them, set them up on your own, or do they provide any help? I understand that people do audition rotations during this time, but only a few, correct?
-is there any electives for 3rd year or they ALL in 4th?
 
1st off its 36 weeks, 16 weeks in medicine, 16 in “surgery” and 4 in special topics. You completely set up your entire 4th year so you could set it up anywhere but this can be difficult for some rotation specialties. Most program use VSAS to set up and approve rotations.
At Carey you are completely on your own, the school only approves or denies your rotations.
Audition rotations continue throughout the entire interview season. As a DO this is almost the only way to get interviews in many specialities anymore.
There is 1 surgery selective and 1 medicine selective in 3rd year.

The way Carey and some other DO school treat 3rd and 4th year puts you at a strange disadvantage with a nice benefit. ( Excluding EM, because they can cheat and name the rotation some thing else and it counts for surgery or medicine) Most DO schools have requirements that have to be done in 4th year that may interfere with early auditions, at Carey you dont. However you are limited in the number of rotations you can do in any single speciality and if the school doesn’t approve it youre are not insured to do it. So if you do it anyway you risk expulsion. At other schools you can audition the entire season, some of these rotations may not count towards graduation but you could still do them or do some required 4th year stuff in your 3rd year so you can do more auditions.

Many programs think you are lazy or not prepared if you do a non interested speciality during interview season. They just are not used to it so they assume you did something wrong.

At this stage in the game it shouldnt matter to you about 3rd and 4th year as everything I just told you will be completely different by the time you get to your 3rd and 4th years.

Things you should be worried about is 3rd year is a preceptor based year meaning weak education with little connections to residency programs and 4th year is completely on your own making you wonder why do you pay the same in tuition?
 
Deferred decision 11/8, interviewed 11/2... Apparently all the seats have been filled, and deferred applicants will be given opportunities as spots open up.
 
I can not speak to ARCOM or most other DO programs, I do know that some have great connections with former Aoa programs and have most of their rotation in those hospitals you can match into and they make it easier to set up. Carey doesn't have that.

I don't believe this statement is true. Merritt Health in Hattiesburg has 2 residency programs(EM and IM) that WCUCOM helped start. This two programs can be considered a home program for Carey and the hospital serves as a core site for 3rd year. There is also FGH hospital which has an FM program that is very Carey and DO friendly. The PD for EM program at Merrit Health is the wife of the dean at Wcuom. You can definitely do your rotations there your 3rd and 4th year.
 
  • Like
Reactions: 1 user
I can not speak to ARCOM or most other DO programs, I do know that some have great connections with former Aoa programs and have most of their rotation in those hospitals you can match into and they make it easier to set up. Carey doesn't have that.

I don't believe this statement is true. Merritt Health in Hattiesburg has 2 residency programs(EM and IM) that WCUCOM helped start. This two programs can be considered a home program for Carey and the hospital serves as a core site for 3rd year. There is also FGH hospital which has an FM program that is very Carey and DO friendly. The PD for EM program at Merrit Health is the wife of the dean at Wcuom. You can definitely do your rotations there your 3rd and 4th year.

Those two hospitals are in no way home programs by any understanding of what a home program is. You can do rotations at these hospitals and you can match in these programs but Carey has no faculty or formal associations with either program. Just having a hospital or two that you could match at in very very limited specialties does not mean your program has a home program nor does it mean your school has associations with any hospital. Most DO school dont have home programs. Some have strong associations with hospitals with broad residency programs, Carey doesnt have either.
They will tell you they have agreements or ties or whatever. If they dont have departments of medicine associated in that hospital where faculty that teach residents and teach medical students are then they dont have a home hospital which mean if you go to that school you dont have that safety net.
Dont let them blur the lines and make you think they have something they dont.
 
  • Like
Reactions: 1 user
Interviewed 11/2. Deferred. Accepted 11/15


Sent from my iPhone using SDN mobile
 
  • Like
Reactions: 2 users
Hi guys, just 2 quick questions.

Is attendance for all lectures mandatory or are just some mandatory and others not?

Are all the lectures recorded so we can watch them again later?

Thanks!
 
Hi guys, just 2 quick questions.

Is attendance for all lectures mandatory or are just some mandatory and others not?

Are all the lectures recorded so we can watch them again later?

Thanks!
There are only 2 classes that officially take attendance in addition to doctoring skills and OPP which has mandatory attendance. All lectures are recorded and the new recording system this year has been great!
 
  • Like
Reactions: 3 users
Hi All,

I graduated Carey this past year and matched with my first choice residency program. I figured I would check in because it’s very rare when graduates post on this thread. Because I no longer have any ties to the program I feel I can speak very openly and freely about my experience.

I am doing this because I wish someone told me this information before I applied to med school and while I was in med school and before I applied to residency. I really hope it helps.

PART ONE of this post is new material since I have graduated which I think would be most applicable to current medical students as they approach the residency applications and beyond.


PART TWO is material I’m rebooting verbatim from a post I made while I was a fourth-year student that I hope will help both getting in and navigating your time in medical school. Caveat part two I’m no longer current student and have no idea what goes on campus today. I can only speak from my historical experience.

Ok here we go, if you’re just applying to med school you may want to scroll to part two...

PART ONE:
As I said above I matched my first choice program. I’m going to add this year I am part of my residency’s selection committee so I’ve been fortunate to see the process from the other side; see the interview applicants, read their personal statements, read their CVs, have input into who gets an interview etc.

My class has a whole, however, had a very poor match rate. At one point around match last year I knew the exact specifics but as they are no longer relevant to me I will speak anecdotally from memory while trying to remain as accurate as possible. My goal with this isn’t to argue statistics but merely to paint an overall picture of what the stats mean.

The numbers are +/- a few as I recall but they’re close enough for this conversation.

We matched approximately 56% of the graduating class (~90, however we began with 116 and gained several more along the way). That included both the AOA and ACGME match in total. In other words and again this is approximate 50/90 graduates actually matched.

Running total 50/90 matched

~18 people were successful in the AOA scramble. To be very clear on what that means these people did not match and had to personally call programs to ask if they had open positions they could fill after the match occurred.

2-3 successfully soap ACGME

Running total ~70/90 matched or scrambled/soap which means they are in the residency program without having to reapply.

~15 people were accepted into TRI (traditional rotating internship) programs. Think of this like the residency version of an MBS program. This means they did not match, they did not successfully scramble AOA or soap ACGME and they have added a year to the residency. TRI does not count as your basic three years of residency. A person in a TRI has to reapply to residency the following year while they are doing their internship. Statistically the odds of matching the second time around are very low. TRI interns are often stigmatized.

Running totals: approximately 85/90 found a home. Remember the people who landed a TRI will have to reapply for residency this year.

The rest will either reapply for residency this year or find job doing something else.

That said, many of my friends received their first choice program or a program somewhere at the top of their list because they were well prepared, well informed and realistic.

These numbers are very poor. It’s very evident there is a disconnect between what some of our graduates wanted and reality. There are a lot of ways to help your application and there a lot of ways to sink your application during this process.

There’s a common theme with many applicants who do not match in that is having unrealistic expectations. Often people apply the programs due to perceived status of a program or field.

When you’re entering this process take a very good look at yourself, your scores, your grades, your school, your preceptor evaluations, related research, perceived interest in the field, how well you are liked by your peers and preceptors and how well you interact with others in general. I promise you every one of these factors will come into play during your applications.

Factor all these together and look at the field and the specific programs to which you are applying. Are you shooting for dermatology or orthopedics which are historically extremely selective residency programs? If you’re board (USMLE) scores are in the 95th percentile along with your grades, you have multiple relevant publications, know someone in the field, and all your preceptors and classmates want to take you out and have a beer with you, then you probably have a pretty good shot. If you’re not checking those boxes, the reality is look for less competitive field or you may be applying again next year.

Don’t apply to a field based on perceived prestige or solely based on how much money you think you will make. I know many attending‘s in pediatrics, family medicine, internal medicine who make every bit as much as an orthopedist because they’re intelligent, use their time wisely and know how to maximize their worth.

Apply to a field because you love the work, because you’re passionate about it, and because you see yourself doing it 30 years from now, otherwise you will be 10 years into your career and hate what you do. The money will work itself out.

Basic tips:

Apply early

Show interest in the field

Be very specific to that field in your personal statement

Have your letters of recommendations specifically talk about that field. Don’t be stupid enough you have your LOR say “he will make a great surgeon” when you’re applying to internal medicine Even if you’re applying to surgery as well.

Don’t wait to take step 2. Plan ahead and get it done early.

Be gracious

Keep in mind your interview begins the moment you set foot in that town until the moment you leave. Every single person you encounter may have an impact on you getting accepted.

Secretaries and program coordinators have huge input and a direct line to the program directors. Irritate them and you have no chance; impressed them and they will speak highly of you.

I can’t believe I have to say this but I’ve seen it happen, don’t be a jerk, don’t brag about yourself and be genuinely interested.

Keep your personal statement to one page. get to the point and keep the readers attention. Most importantly it should relate to why you connect with the field to which you are applying.

Do something meaningful for your CV. Do something of meaning in your community or publish something or at least put something on there that’s interesting (run a marathon, climb a mountain, bungee jump over the moon whatever just make it memorable) No one cares that you were third vice chair in your med school club X. Don’t get me wrong, it’s very good to show specific interest in the club to the field you’re applying but don’t make that the centerpiece of your application.

Be thoughtful after your interview and send a thank you card or a thank you email to everyone who interviewed you, the program director, the program coordinator. Trust me it goes a long way.


PART TWO

There have been many repeat questions from new interviewees or newly accepted students. I am going to partially recycle a post I made a few months back with a lot of new updates and info. I have tried to make this post easy to scroll through its crazy long, I addedheadings by timeline year by year as follows:

Q&A – short and to the point
Premed - detailed
OMS1- detailed
OMS2- detailed
OMS3- detailed
OMS4 - detailed
I have been as unbiased as I can, based on my experience, successes and mistakes. I truly wish you all the best.

Context: This is my SDN swansong. I am a fourth year and recently matched my first-choice program. It felt REALLY GOOD to see DO next to my name in the residency contract! Here is my experience and perspective.

  • Q&A
Premed/OMS1 Q&A
Carey vs MD schools?
MD 100% see premed below.

Carey vs Caribbean schools?
Carey 100% see premed below.

Carey vs other DO schools?
Too much arguing on this topic on this thread. ACGME doesn’t care; seriously they don’t care if you’re from Carey or Touro, PCOM, TCOM, ZCOM, QCOM or wherever, unless the program is located by the specific DO school and they interact with the students on rotation. No ACGME program where I interviewed knew anything about Carey good or bad. They lumped all DO schools into the same general category and don’t know enough about them to stratify schools. See last few paragraphs in premed below.

Is Carey geared towards primary care?
Absolutely, its in the mission statement.

Is the attendance policy excessive?
Yes, according to most, but its there. Work around it.

Can I do 4th year away rotations away or in Hattiesburg?
Yes, and yes, but you need to plan them yourself. See OMS4 above

Best part about Carey?
Students 100%, also the newer, younger faculty seems to be pretty invested in the students.

Worst part?
Red tape all 4 years.

Best (most relevant to intended goal) educational aspect of Carey?
COMLEX level 2 PE prep.

Worst?
USMLE 1 or 2 anything

What is OMS1 or 2 like?
Ask an OMS1 or 2, its changed dramatically since my class.

Can I become a great Doctor after graduating Carey?
100%

OMS2-4 Q&A

Is there OMT work in OMS3 and 4?
Yes, to the very last OMS4 rotation.

Do I have to perform OMT on Rotations?
No

Should I take the USMLE?
Yes, if you want anything in ACGME other than FM or IM.

Is there an MD bias?
Not that I ever saw. Competent is competent. This seems to be a premed construct.

ACGME or AOA?
If you are class of 2019, IMO 100% ACGME if you can, IMO.
2020, this is a problem that will not exist, but the competition will no doubt increase because many AOA spots are going by the wayside.

Will I get much support with my application to residency?
No. I believe it will improve in the next few years though.

Has attending Carey helped or hurt my match?
UGH. I, along with many are very happy with match, but many are not. See OMS4 paragraphs below.

What about uber competitive specialties ex. ENT, derm etc.?
Unlikely based on past stats. If you want Derm, etc. go to a top 40 NIH funded MD school, kill the boards AND have a family member or close friend in that field. They are crazy hard to get no matter where you go!
Based on this year, if you apply from here 95%+ of you will get FM, IM, Peds, Psych, OB, ER, Gen Surg, Rads, Gas in that or similar order. There were a couple outliers this year, but this is what you can expect.

Will it help my match chances to apply in my home state?
Yes 100%

What is the 2018 placement / match rate this year?
Placement rate and match rate are vastly different things. Placement also includes AOA scramble, ACGME SOAP and post SOAP AOA empty spots, TRIs etc.
Placement rate TBD.
AOA match by school link
2018 Summary by College

Those numbers are convoluted, what does the above link mean?
Yep, that’s AOA. Basically, WCU matched similarly to the other DO schools in the AOA match.

What was the 2018 ACGME match rate?
Anecdotal at this point, nothing released by the school yet.

NRMP match link
http://www.nrmp.org/wp-content/uploads/2018/02/Results-and-Data-SMS-2018.pdf

What can I do to have the best match from Carey?
Work hard, make connections, do well on boards, be realistic about your field based on stats, be really cool to your attendings, residents, and other med students on rotations and interviews. Don’t brown nose or act like you know more than you do, it irritates PDs and attendings. Be willing to learn from anyone.

Detailed Info by year:
  • Premed students:
No offense meant, but you truly have no idea yet. Knowing undergard or grad level biochem and cell bio is a tiny fraction of med school. This is likely the single biggest decision of your life which will impact you for the next 30+ years. Listen to a wide array of people who have been through it and plan accordingly. There is a spectrum of people in med school and more specifically at Carey ranging from completely miserable to reasonably content.

I don’t remember anyone doing backflips down the hall in OMS1 or 2.

OMS3 is pretty good

4th year is awesome!

Getting in: Take all the interviews you can until you get an acceptance (or 2), then you can afford to be choosey. IMO I would NOT forego an acceptance ANYWHERE to do an MBS or the like. There is no point in adding a year of school, debt etc. to maybe have a shot at a place you may or may not like better. This process is a grind; take the bird in hand.

Getting into Carey: If you get an interview you have a great chance to be accepted unless you completely blow the interview which is very unlikely If you are waitlisted, hang in there, you still have a shot. In my first year they added people from the list until the second day of classes.

Carey vs other DO schools vs MD vs Caribbean (classic SDN debate):

In truth I hated the first two preclinical years at Carey, there are plenty of specifics as to why by other students in this thread. After two years of rotations / residency interviews meeting other DO, MD, and IMG grads, no one has told me their first two years of med school were fantastic. Many if not most of the things that infuriate students at Carey exist at the other DO schools as well.

I had several options. I chose Carey because of the students. Honestly, if I did not have the friends I made at Carey, med school would have been a lot harder. There are gunners at every school, but I think we have fewer than most.

RE: Experiences of My MD friends

Less frustration in the preclinical years

Better environment to succeed on the boards. That statement is not speculative, I have seen course materials and syllabi from friends, and it is better. MD programs have had more time to ferret out things that do not benefit the students and typically are set in established academic environments, whereas DO schools are almost universally newer and still figuring out what works. Also, keep in mind we spend 20%+ of our time on OMT material which never appears on USMLE, theoretically giving them less material to study in the same time frame.

Better opportunities to match in highly competitive fields-see above + agreements with academic hospital systems, more help from admin, advisors, match stats, connections, etc.

MD students generally have less hands-on experience in years 3-4 because there are multiple layers of students, interns, residents, fellows, etc. above them getting in on procedures and patient care.
I believe the relationships a student can make with a preceptor at Carey is one of the better selling points of the school. My LORs to ERAS were outstanding, and even better my preceptors discussed them with me prior to submission. Without question, they opened doors for me to residency interviews I may have not had if I only worked with residents and had cookie cutter fill in the blank LORs.

The IMG grads (Caribbean or elsewhere) I have met have a rough go all around. DON’T DO IT UNLESS YOU HAVE NO OTHER OPTION!

They are basically on their own and self-study for the first two years and then struggle to get rotations in the U.S. I met an IMG MD at one of my interviews who was in her third application cycle and working as a nurse to pay loans until she found a residency. Access to good preceptors is extremely important along with a chance at an AI (acting internship) to get the attention of a specific program. The students in the Caribbean are encouraged to apply to 150-250 programs to try and get a residency spot. Again, not speculation but information offered by a Ross grad during an interview dinner. The IMGs I know lumped U.S. grads together, MD and DO when they compared their experience to ours.

  • OMS1

Goal: make it unscathed to OMS2.

Don’t get caught up in the med school frenzy, politics or the like. There is plenty of time for you to worry about residency, boards etc. Grind, study your power points, do as well as you can in class to lay a framework of knowledge onto which you will add to over the next 2 years. There are many things which will infuriate you but don’t let it get to you. Getting angry with things admin does will not help you nor will it change their outlook or position. It’s going to bring you down and your relationships with your studies, family, SOs, and friends will suffer if you’re irritated from things beyond your control. You are here; suck it up and remember you will soon be a doctor. Things will get better, I promise.

Enjoy your summer, maybe do some research if you can.

  • OMS2
Goal: don’t get tripped up by a class and work hard for Step/Level 1

You know how the classes work by now. Mind your attendance so you don’t fail but do what is best for your board studies. If you don’t learn from them in class, duck out and make better use of your time (while still maintaining 80%). Use your most precious commodity (time) wisely. Don’t sit back in class playing games or watching Netflix and then complain about barely passing or poor board scores. You will regret it at match time. Keep grinding and do things outside of school when you can to preserve your sanity.

UFAP (uworld first aid pathoma) or something similar. Pick a boards program and stick with it. Everybody and their brother is going to try and sell you something this year, emails, classmates trying to rep a program for a discount, sales people coming to school, you will see it all. Its all expensive. Everyone learns in their own way, looking back I used too many resources and spread myself too thin among them. I probably could have squeezed out more points by sticking with the tried and true resources. Keep in mind to do COMLEX style questions as well. It is a terribly written, vague exam but you MUST pass it.

Make sure you pass the COMSAE!!! It will mess up your study schedule if you don’t. They will probably force you to take some type of school sponsored course which is unlikely to help you as an individual. Be strategic about when you are taking step/level 1.

Begin to think about a field of interest but don’t fixate because it may change once you get into rotations.

  • OMS3
Goal: Shine on rotations and get those LORs, and work hard for step 2

There are many great resources for you, onlinemeded is fantastic, FA

Figure out what you want to do when you grow up and explore that field with your electives.

MAKE CONNECTIONS where you can, it’s going to get real, and very soon. Get those VSAS apps out ASAP. The AI spots fill up fast (don’t call it an “audition rotation”, It makes you sound less informed on how the system works). ACGME calls it acting internship (AI) or externship or sub-intership (Sub-I). There are also many good non-VSAS programs including FGH in Hattiesburg. In either case book the AI early.

Boards

Remember what worked or didn’t work for you on step/level 1. No reason to reinvent the wheel. And yes, COMLEX 2 is also terribly written.

Remember you must come back for OMT or OSCE which will take a big chunk away from step/level 2 studies. Plan and schedule accordingly.

The PE: its easy. Don’t stress it. If you have done well on your rotations, it will be fine. Learn how to craft a good SOAP note, you are going to need it for many years to come. Probably the best thing Carey does is prep you for the PE. Its shockingly similar. A+ for them on this one.

  • OMS4
Get AIs and WORK HARD!!!!!!!!!!!!!!!!!!!!!!!!!!! I cannot stress this enough. If you are borderline on paper but kill the rotation. it may be enough to get you that residency spot. If you are great on paper but flub the rotation, you are done. Anecdotally, I have seen it work both ways.

Be fully aware of what you will need for your applications, ex. SLOEs for EM are due by September 15th and plan ahead.

Get your application into ERAS on time!!!The Dean’s letter (or MPSE) comes out in October. Don’t wait for it. Most of your interviews will be booked long before it comes out if you are on your game.

Recognize your strengths and weaknesses and be realistic about your chances; seriously, you need to do this. Are you mediocre or not so great on paper? Do you get under people’s skin? Are your preceptor reviews less than stellar?

Most people who matched in my class did this. Many people who did not match were often unrealistic with their chances into their applied field or program, conversely, there were a few in our class who were tailor made for their field and applied appropriately and unfortunately, it didn’t work out for them. That’s not to say don’t shoot for the moon but have a plan B just in case. There are alternative paths if you do not match ex. TRI.

Remember that when you get to this point it is a job. Refer to the program director survey on the ACGME website to see what is important in obtaining a residency.

Among ALL specialties factors and importance for ranking applicants post interview # 1-4 have nothing to do with scores/ class rank (link below). I am by no means diminishing the importance of board scores. They are very important, and extremely important for a highly competitive specialty. But, keep in mind PD (program directors), faculty and residents want to know if they can work with you for long hours under high stress for 3-5 years straight. Being a decent and modest student/ person and having genuine interest in the field will serve you well in your journey.

http://www.nrmp.org/wp-content/uploads/2016/09/NRMP-2016-Program-Director-Survey.pdf

Good luck to all!
 
  • Like
Reactions: 20 users
How long does it typically take for them to send secondary apps once primary has been submitted?
 
Hi All,

I graduated Carey this past year and matched with my first choice residency program. I figured I would check in because it’s very rare when graduates post on this thread. Because I no longer have any ties to the program I feel I can speak very openly and freely about my experience.

I am doing this because I wish someone told me this information before I applied to med school and while I was in med school and before I applied to residency. I really hope it helps.

PART ONE of this post is new material since I have graduated which I think would be most applicable to current medical students as they approach the residency applications and beyond.


PART TWO is material I’m rebooting verbatim from a post I made while I was a fourth-year student that I hope will help both getting in and navigating your time in medical school. Caveat part two I’m no longer current student and have no idea what goes on campus today. I can only speak from my historical experience.

Ok here we go, if you’re just applying to med school you may want to scroll to part two...

PART ONE:
As I said above I matched my first choice program. I’m going to add this year I am part of my residency’s selection committee so I’ve been fortunate to see the process from the other side; see the interview applicants, read their personal statements, read their CVs, have input into who gets an interview etc.

My class has a whole, however, had a very poor match rate. At one point around match last year I knew the exact specifics but as they are no longer relevant to me I will speak anecdotally from memory while trying to remain as accurate as possible. My goal with this isn’t to argue statistics but merely to paint an overall picture of what the stats mean.

The numbers are +/- a few as I recall but they’re close enough for this conversation.

We matched approximately 56% of the graduating class (~90, however we began with 116 and gained several more along the way). That included both the AOA and ACGME match in total. In other words and again this is approximate 50/90 graduates actually matched.

Running total 50/90 matched

~18 people were successful in the AOA scramble. To be very clear on what that means these people did not match and had to personally call programs to ask if they had open positions they could fill after the match occurred.

2-3 successfully soap ACGME

Running total ~70/90 matched or scrambled/soap which means they are in the residency program without having to reapply.

~15 people were accepted into TRI (traditional rotating internship) programs. Think of this like the residency version of an MBS program. This means they did not match, they did not successfully scramble AOA or soap ACGME and they have added a year to the residency. TRI does not count as your basic three years of residency. A person in a TRI has to reapply to residency the following year while they are doing their internship. Statistically the odds of matching the second time around are very low. TRI interns are often stigmatized.

Running totals: approximately 85/90 found a home. Remember the people who landed a TRI will have to reapply for residency this year.

The rest will either reapply for residency this year or find job doing something else.

That said, many of my friends received their first choice program or a program somewhere at the top of their list because they were well prepared, well informed and realistic.

These numbers are very poor. It’s very evident there is a disconnect between what some of our graduates wanted and reality. There are a lot of ways to help your application and there a lot of ways to sink your application during this process.

There’s a common theme with many applicants who do not match in that is having unrealistic expectations. Often people apply the programs due to perceived status of a program or field.

When you’re entering this process take a very good look at yourself, your scores, your grades, your school, your preceptor evaluations, related research, perceived interest in the field, how well you are liked by your peers and preceptors and how well you interact with others in general. I promise you every one of these factors will come into play during your applications.

Factor all these together and look at the field and the specific programs to which you are applying. Are you shooting for dermatology or orthopedics which are historically extremely selective residency programs? If you’re board (USMLE) scores are in the 95th percentile along with your grades, you have multiple relevant publications, know someone in the field, and all your preceptors and classmates want to take you out and have a beer with you, then you probably have a pretty good shot. If you’re not checking those boxes, the reality is look for less competitive field or you may be applying again next year.

Don’t apply to a field based on perceived prestige or solely based on how much money you think you will make. I know many attending‘s in pediatrics, family medicine, internal medicine who make every bit as much as an orthopedist because they’re intelligent, use their time wisely and know how to maximize their worth.

Apply to a field because you love the work, because you’re passionate about it, and because you see yourself doing it 30 years from now, otherwise you will be 10 years into your career and hate what you do. The money will work itself out.

Basic tips:

Apply early

Show interest in the field

Be very specific to that field in your personal statement

Have your letters of recommendations specifically talk about that field. Don’t be stupid enough you have your LOR say “he will make a great surgeon” when you’re applying to internal medicine Even if you’re applying to surgery as well.

Don’t wait to take step 2. Plan ahead and get it done early.

Be gracious

Keep in mind your interview begins the moment you set foot in that town until the moment you leave. Every single person you encounter may have an impact on you getting accepted.

Secretaries and program coordinators have huge input and a direct line to the program directors. Irritate them and you have no chance; impressed them and they will speak highly of you.

I can’t believe I have to say this but I’ve seen it happen, don’t be a jerk, don’t brag about yourself and be genuinely interested.

Keep your personal statement to one page. get to the point and keep the readers attention. Most importantly it should relate to why you connect with the field to which you are applying.

Do something meaningful for your CV. Do something of meaning in your community or publish something or at least put something on there that’s interesting (run a marathon, climb a mountain, bungee jump over the moon whatever just make it memorable) No one cares that you were third vice chair in your med school club X. Don’t get me wrong, it’s very good to show specific interest in the club to the field you’re applying but don’t make that the centerpiece of your application.

Be thoughtful after your interview and send a thank you card or a thank you email to everyone who interviewed you, the program director, the program coordinator. Trust me it goes a long way.


PART TWO

There have been many repeat questions from new interviewees or newly accepted students. I am going to partially recycle a post I made a few months back with a lot of new updates and info. I have tried to make this post easy to scroll through its crazy long, I addedheadings by timeline year by year as follows:

Q&A – short and to the point
Premed - detailed
OMS1- detailed
OMS2- detailed
OMS3- detailed
OMS4 - detailed
I have been as unbiased as I can, based on my experience, successes and mistakes. I truly wish you all the best.

Context: This is my SDN swansong. I am a fourth year and recently matched my first-choice program. It felt REALLY GOOD to see DO next to my name in the residency contract! Here is my experience and perspective.

  • Q&A
Premed/OMS1 Q&A
Carey vs MD schools?
MD 100% see premed below.

Carey vs Caribbean schools?
Carey 100% see premed below.

Carey vs other DO schools?
Too much arguing on this topic on this thread. ACGME doesn’t care; seriously they don’t care if you’re from Carey or Touro, PCOM, TCOM, ZCOM, QCOM or wherever, unless the program is located by the specific DO school and they interact with the students on rotation. No ACGME program where I interviewed knew anything about Carey good or bad. They lumped all DO schools into the same general category and don’t know enough about them to stratify schools. See last few paragraphs in premed below.

Is Carey geared towards primary care?
Absolutely, its in the mission statement.

Is the attendance policy excessive?
Yes, according to most, but its there. Work around it.

Can I do 4th year away rotations away or in Hattiesburg?
Yes, and yes, but you need to plan them yourself. See OMS4 above

Best part about Carey?
Students 100%, also the newer, younger faculty seems to be pretty invested in the students.

Worst part?
Red tape all 4 years.

Best (most relevant to intended goal) educational aspect of Carey?
COMLEX level 2 PE prep.

Worst?
USMLE 1 or 2 anything

What is OMS1 or 2 like?
Ask an OMS1 or 2, its changed dramatically since my class.

Can I become a great Doctor after graduating Carey?
100%

OMS2-4 Q&A

Is there OMT work in OMS3 and 4?
Yes, to the very last OMS4 rotation.

Do I have to perform OMT on Rotations?
No

Should I take the USMLE?
Yes, if you want anything in ACGME other than FM or IM.

Is there an MD bias?
Not that I ever saw. Competent is competent. This seems to be a premed construct.

ACGME or AOA?
If you are class of 2019, IMO 100% ACGME if you can, IMO.
2020, this is a problem that will not exist, but the competition will no doubt increase because many AOA spots are going by the wayside.

Will I get much support with my application to residency?
No. I believe it will improve in the next few years though.

Has attending Carey helped or hurt my match?
UGH. I, along with many are very happy with match, but many are not. See OMS4 paragraphs below.

What about uber competitive specialties ex. ENT, derm etc.?
Unlikely based on past stats. If you want Derm, etc. go to a top 40 NIH funded MD school, kill the boards AND have a family member or close friend in that field. They are crazy hard to get no matter where you go!
Based on this year, if you apply from here 95%+ of you will get FM, IM, Peds, Psych, OB, ER, Gen Surg, Rads, Gas in that or similar order. There were a couple outliers this year, but this is what you can expect.

Will it help my match chances to apply in my home state?
Yes 100%

What is the 2018 placement / match rate this year?
Placement rate and match rate are vastly different things. Placement also includes AOA scramble, ACGME SOAP and post SOAP AOA empty spots, TRIs etc.
Placement rate TBD.
AOA match by school link
2018 Summary by College

Those numbers are convoluted, what does the above link mean?
Yep, that’s AOA. Basically, WCU matched similarly to the other DO schools in the AOA match.

What was the 2018 ACGME match rate?
Anecdotal at this point, nothing released by the school yet.

NRMP match link
http://www.nrmp.org/wp-content/uploads/2018/02/Results-and-Data-SMS-2018.pdf

What can I do to have the best match from Carey?
Work hard, make connections, do well on boards, be realistic about your field based on stats, be really cool to your attendings, residents, and other med students on rotations and interviews. Don’t brown nose or act like you know more than you do, it irritates PDs and attendings. Be willing to learn from anyone.

Detailed Info by year:
  • Premed students:
No offense meant, but you truly have no idea yet. Knowing undergard or grad level biochem and cell bio is a tiny fraction of med school. This is likely the single biggest decision of your life which will impact you for the next 30+ years. Listen to a wide array of people who have been through it and plan accordingly. There is a spectrum of people in med school and more specifically at Carey ranging from completely miserable to reasonably content.

I don’t remember anyone doing backflips down the hall in OMS1 or 2.

OMS3 is pretty good

4th year is awesome!

Getting in: Take all the interviews you can until you get an acceptance (or 2), then you can afford to be choosey. IMO I would NOT forego an acceptance ANYWHERE to do an MBS or the like. There is no point in adding a year of school, debt etc. to maybe have a shot at a place you may or may not like better. This process is a grind; take the bird in hand.

Getting into Carey: If you get an interview you have a great chance to be accepted unless you completely blow the interview which is very unlikely If you are waitlisted, hang in there, you still have a shot. In my first year they added people from the list until the second day of classes.

Carey vs other DO schools vs MD vs Caribbean (classic SDN debate):

In truth I hated the first two preclinical years at Carey, there are plenty of specifics as to why by other students in this thread. After two years of rotations / residency interviews meeting other DO, MD, and IMG grads, no one has told me their first two years of med school were fantastic. Many if not most of the things that infuriate students at Carey exist at the other DO schools as well.

I had several options. I chose Carey because of the students. Honestly, if I did not have the friends I made at Carey, med school would have been a lot harder. There are gunners at every school, but I think we have fewer than most.

RE: Experiences of My MD friends

Less frustration in the preclinical years

Better environment to succeed on the boards. That statement is not speculative, I have seen course materials and syllabi from friends, and it is better. MD programs have had more time to ferret out things that do not benefit the students and typically are set in established academic environments, whereas DO schools are almost universally newer and still figuring out what works. Also, keep in mind we spend 20%+ of our time on OMT material which never appears on USMLE, theoretically giving them less material to study in the same time frame.

Better opportunities to match in highly competitive fields-see above + agreements with academic hospital systems, more help from admin, advisors, match stats, connections, etc.

MD students generally have less hands-on experience in years 3-4 because there are multiple layers of students, interns, residents, fellows, etc. above them getting in on procedures and patient care.
I believe the relationships a student can make with a preceptor at Carey is one of the better selling points of the school. My LORs to ERAS were outstanding, and even better my preceptors discussed them with me prior to submission. Without question, they opened doors for me to residency interviews I may have not had if I only worked with residents and had cookie cutter fill in the blank LORs.

The IMG grads (Caribbean or elsewhere) I have met have a rough go all around. DON’T DO IT UNLESS YOU HAVE NO OTHER OPTION!

They are basically on their own and self-study for the first two years and then struggle to get rotations in the U.S. I met an IMG MD at one of my interviews who was in her third application cycle and working as a nurse to pay loans until she found a residency. Access to good preceptors is extremely important along with a chance at an AI (acting internship) to get the attention of a specific program. The students in the Caribbean are encouraged to apply to 150-250 programs to try and get a residency spot. Again, not speculation but information offered by a Ross grad during an interview dinner. The IMGs I know lumped U.S. grads together, MD and DO when they compared their experience to ours.

  • OMS1

Goal: make it unscathed to OMS2.

Don’t get caught up in the med school frenzy, politics or the like. There is plenty of time for you to worry about residency, boards etc. Grind, study your power points, do as well as you can in class to lay a framework of knowledge onto which you will add to over the next 2 years. There are many things which will infuriate you but don’t let it get to you. Getting angry with things admin does will not help you nor will it change their outlook or position. It’s going to bring you down and your relationships with your studies, family, SOs, and friends will suffer if you’re irritated from things beyond your control. You are here; suck it up and remember you will soon be a doctor. Things will get better, I promise.

Enjoy your summer, maybe do some research if you can.

  • OMS2
Goal: don’t get tripped up by a class and work hard for Step/Level 1

You know how the classes work by now. Mind your attendance so you don’t fail but do what is best for your board studies. If you don’t learn from them in class, duck out and make better use of your time (while still maintaining 80%). Use your most precious commodity (time) wisely. Don’t sit back in class playing games or watching Netflix and then complain about barely passing or poor board scores. You will regret it at match time. Keep grinding and do things outside of school when you can to preserve your sanity.

UFAP (uworld first aid pathoma) or something similar. Pick a boards program and stick with it. Everybody and their brother is going to try and sell you something this year, emails, classmates trying to rep a program for a discount, sales people coming to school, you will see it all. Its all expensive. Everyone learns in their own way, looking back I used too many resources and spread myself too thin among them. I probably could have squeezed out more points by sticking with the tried and true resources. Keep in mind to do COMLEX style questions as well. It is a terribly written, vague exam but you MUST pass it.

Make sure you pass the COMSAE!!! It will mess up your study schedule if you don’t. They will probably force you to take some type of school sponsored course which is unlikely to help you as an individual. Be strategic about when you are taking step/level 1.

Begin to think about a field of interest but don’t fixate because it may change once you get into rotations.

  • OMS3
Goal: Shine on rotations and get those LORs, and work hard for step 2

There are many great resources for you, onlinemeded is fantastic, FA

Figure out what you want to do when you grow up and explore that field with your electives.

MAKE CONNECTIONS where you can, it’s going to get real, and very soon. Get those VSAS apps out ASAP. The AI spots fill up fast (don’t call it an “audition rotation”, It makes you sound less informed on how the system works). ACGME calls it acting internship (AI) or externship or sub-intership (Sub-I). There are also many good non-VSAS programs including FGH in Hattiesburg. In either case book the AI early.

Boards

Remember what worked or didn’t work for you on step/level 1. No reason to reinvent the wheel. And yes, COMLEX 2 is also terribly written.

Remember you must come back for OMT or OSCE which will take a big chunk away from step/level 2 studies. Plan and schedule accordingly.

The PE: its easy. Don’t stress it. If you have done well on your rotations, it will be fine. Learn how to craft a good SOAP note, you are going to need it for many years to come. Probably the best thing Carey does is prep you for the PE. Its shockingly similar. A+ for them on this one.

  • OMS4
Get AIs and WORK HARD!!!!!!!!!!!!!!!!!!!!!!!!!!! I cannot stress this enough. If you are borderline on paper but kill the rotation. it may be enough to get you that residency spot. If you are great on paper but flub the rotation, you are done. Anecdotally, I have seen it work both ways.

Be fully aware of what you will need for your applications, ex. SLOEs for EM are due by September 15th and plan ahead.

Get your application into ERAS on time!!!The Dean’s letter (or MPSE) comes out in October. Don’t wait for it. Most of your interviews will be booked long before it comes out if you are on your game.

Recognize your strengths and weaknesses and be realistic about your chances; seriously, you need to do this. Are you mediocre or not so great on paper? Do you get under people’s skin? Are your preceptor reviews less than stellar?

Most people who matched in my class did this. Many people who did not match were often unrealistic with their chances into their applied field or program, conversely, there were a few in our class who were tailor made for their field and applied appropriately and unfortunately, it didn’t work out for them. That’s not to say don’t shoot for the moon but have a plan B just in case. There are alternative paths if you do not match ex. TRI.

Remember that when you get to this point it is a job. Refer to the program director survey on the ACGME website to see what is important in obtaining a residency.

Among ALL specialties factors and importance for ranking applicants post interview # 1-4 have nothing to do with scores/ class rank (link below). I am by no means diminishing the importance of board scores. They are very important, and extremely important for a highly competitive specialty. But, keep in mind PD (program directors), faculty and residents want to know if they can work with you for long hours under high stress for 3-5 years straight. Being a decent and modest student/ person and having genuine interest in the field will serve you well in your journey.

http://www.nrmp.org/wp-content/uploads/2016/09/NRMP-2016-Program-Director-Survey.pdf

Good luck to all!
I like that part about the PE prep!
 
  • Like
Reactions: 1 user
Hi All,

I graduated Carey this past year and matched with my first choice residency program. I figured I would check in because it’s very rare when graduates post on this thread. Because I no longer have any ties to the program I feel I can speak very openly and freely about my experience.

I am doing this because I wish someone told me this information before I applied to med school and while I was in med school and before I applied to residency. I really hope it helps.

PART ONE of this post is new material since I have graduated which I think would be most applicable to current medical students as they approach the residency applications and beyond.


PART TWO is material I’m rebooting verbatim from a post I made while I was a fourth-year student that I hope will help both getting in and navigating your time in medical school. Caveat part two I’m no longer current student and have no idea what goes on campus today. I can only speak from my historical experience.

Ok here we go, if you’re just applying to med school you may want to scroll to part two...

PART ONE:
As I said above I matched my first choice program. I’m going to add this year I am part of my residency’s selection committee so I’ve been fortunate to see the process from the other side; see the interview applicants, read their personal statements, read their CVs, have input into who gets an interview etc.

My class has a whole, however, had a very poor match rate. At one point around match last year I knew the exact specifics but as they are no longer relevant to me I will speak anecdotally from memory while trying to remain as accurate as possible. My goal with this isn’t to argue statistics but merely to paint an overall picture of what the stats mean.

The numbers are +/- a few as I recall but they’re close enough for this conversation.

We matched approximately 56% of the graduating class (~90, however we began with 116 and gained several more along the way). That included both the AOA and ACGME match in total. In other words and again this is approximate 50/90 graduates actually matched.

Running total 50/90 matched

~18 people were successful in the AOA scramble. To be very clear on what that means these people did not match and had to personally call programs to ask if they had open positions they could fill after the match occurred.

2-3 successfully soap ACGME

Running total ~70/90 matched or scrambled/soap which means they are in the residency program without having to reapply.

~15 people were accepted into TRI (traditional rotating internship) programs. Think of this like the residency version of an MBS program. This means they did not match, they did not successfully scramble AOA or soap ACGME and they have added a year to the residency. TRI does not count as your basic three years of residency. A person in a TRI has to reapply to residency the following year while they are doing their internship. Statistically the odds of matching the second time around are very low. TRI interns are often stigmatized.

Running totals: approximately 85/90 found a home. Remember the people who landed a TRI will have to reapply for residency this year.

The rest will either reapply for residency this year or find job doing something else.

That said, many of my friends received their first choice program or a program somewhere at the top of their list because they were well prepared, well informed and realistic.

These numbers are very poor. It’s very evident there is a disconnect between what some of our graduates wanted and reality. There are a lot of ways to help your application and there a lot of ways to sink your application during this process.

There’s a common theme with many applicants who do not match in that is having unrealistic expectations. Often people apply the programs due to perceived status of a program or field.

When you’re entering this process take a very good look at yourself, your scores, your grades, your school, your preceptor evaluations, related research, perceived interest in the field, how well you are liked by your peers and preceptors and how well you interact with others in general. I promise you every one of these factors will come into play during your applications.

Factor all these together and look at the field and the specific programs to which you are applying. Are you shooting for dermatology or orthopedics which are historically extremely selective residency programs? If you’re board (USMLE) scores are in the 95th percentile along with your grades, you have multiple relevant publications, know someone in the field, and all your preceptors and classmates want to take you out and have a beer with you, then you probably have a pretty good shot. If you’re not checking those boxes, the reality is look for less competitive field or you may be applying again next year.

Don’t apply to a field based on perceived prestige or solely based on how much money you think you will make. I know many attending‘s in pediatrics, family medicine, internal medicine who make every bit as much as an orthopedist because they’re intelligent, use their time wisely and know how to maximize their worth.

Apply to a field because you love the work, because you’re passionate about it, and because you see yourself doing it 30 years from now, otherwise you will be 10 years into your career and hate what you do. The money will work itself out.

Basic tips:

Apply early

Show interest in the field

Be very specific to that field in your personal statement

Have your letters of recommendations specifically talk about that field. Don’t be stupid enough you have your LOR say “he will make a great surgeon” when you’re applying to internal medicine Even if you’re applying to surgery as well.

Don’t wait to take step 2. Plan ahead and get it done early.

Be gracious

Keep in mind your interview begins the moment you set foot in that town until the moment you leave. Every single person you encounter may have an impact on you getting accepted.

Secretaries and program coordinators have huge input and a direct line to the program directors. Irritate them and you have no chance; impressed them and they will speak highly of you.

I can’t believe I have to say this but I’ve seen it happen, don’t be a jerk, don’t brag about yourself and be genuinely interested.

Keep your personal statement to one page. get to the point and keep the readers attention. Most importantly it should relate to why you connect with the field to which you are applying.

Do something meaningful for your CV. Do something of meaning in your community or publish something or at least put something on there that’s interesting (run a marathon, climb a mountain, bungee jump over the moon whatever just make it memorable) No one cares that you were third vice chair in your med school club X. Don’t get me wrong, it’s very good to show specific interest in the club to the field you’re applying but don’t make that the centerpiece of your application.

Be thoughtful after your interview and send a thank you card or a thank you email to everyone who interviewed you, the program director, the program coordinator. Trust me it goes a long way.


PART TWO

There have been many repeat questions from new interviewees or newly accepted students. I am going to partially recycle a post I made a few months back with a lot of new updates and info. I have tried to make this post easy to scroll through its crazy long, I addedheadings by timeline year by year as follows:

Q&A – short and to the point
Premed - detailed
OMS1- detailed
OMS2- detailed
OMS3- detailed
OMS4 - detailed
I have been as unbiased as I can, based on my experience, successes and mistakes. I truly wish you all the best.

Context: This is my SDN swansong. I am a fourth year and recently matched my first-choice program. It felt REALLY GOOD to see DO next to my name in the residency contract! Here is my experience and perspective.

  • Q&A
Premed/OMS1 Q&A
Carey vs MD schools?
MD 100% see premed below.

Carey vs Caribbean schools?
Carey 100% see premed below.

Carey vs other DO schools?
Too much arguing on this topic on this thread. ACGME doesn’t care; seriously they don’t care if you’re from Carey or Touro, PCOM, TCOM, ZCOM, QCOM or wherever, unless the program is located by the specific DO school and they interact with the students on rotation. No ACGME program where I interviewed knew anything about Carey good or bad. They lumped all DO schools into the same general category and don’t know enough about them to stratify schools. See last few paragraphs in premed below.

Is Carey geared towards primary care?
Absolutely, its in the mission statement.

Is the attendance policy excessive?
Yes, according to most, but its there. Work around it.

Can I do 4th year away rotations away or in Hattiesburg?
Yes, and yes, but you need to plan them yourself. See OMS4 above

Best part about Carey?
Students 100%, also the newer, younger faculty seems to be pretty invested in the students.

Worst part?
Red tape all 4 years.

Best (most relevant to intended goal) educational aspect of Carey?
COMLEX level 2 PE prep.

Worst?
USMLE 1 or 2 anything

What is OMS1 or 2 like?
Ask an OMS1 or 2, its changed dramatically since my class.

Can I become a great Doctor after graduating Carey?
100%

OMS2-4 Q&A

Is there OMT work in OMS3 and 4?
Yes, to the very last OMS4 rotation.

Do I have to perform OMT on Rotations?
No

Should I take the USMLE?
Yes, if you want anything in ACGME other than FM or IM.

Is there an MD bias?
Not that I ever saw. Competent is competent. This seems to be a premed construct.

ACGME or AOA?
If you are class of 2019, IMO 100% ACGME if you can, IMO.
2020, this is a problem that will not exist, but the competition will no doubt increase because many AOA spots are going by the wayside.

Will I get much support with my application to residency?
No. I believe it will improve in the next few years though.

Has attending Carey helped or hurt my match?
UGH. I, along with many are very happy with match, but many are not. See OMS4 paragraphs below.

What about uber competitive specialties ex. ENT, derm etc.?
Unlikely based on past stats. If you want Derm, etc. go to a top 40 NIH funded MD school, kill the boards AND have a family member or close friend in that field. They are crazy hard to get no matter where you go!
Based on this year, if you apply from here 95%+ of you will get FM, IM, Peds, Psych, OB, ER, Gen Surg, Rads, Gas in that or similar order. There were a couple outliers this year, but this is what you can expect.

Will it help my match chances to apply in my home state?
Yes 100%

What is the 2018 placement / match rate this year?
Placement rate and match rate are vastly different things. Placement also includes AOA scramble, ACGME SOAP and post SOAP AOA empty spots, TRIs etc.
Placement rate TBD.
AOA match by school link
2018 Summary by College

Those numbers are convoluted, what does the above link mean?
Yep, that’s AOA. Basically, WCU matched similarly to the other DO schools in the AOA match.

What was the 2018 ACGME match rate?
Anecdotal at this point, nothing released by the school yet.

NRMP match link
http://www.nrmp.org/wp-content/uploads/2018/02/Results-and-Data-SMS-2018.pdf

What can I do to have the best match from Carey?
Work hard, make connections, do well on boards, be realistic about your field based on stats, be really cool to your attendings, residents, and other med students on rotations and interviews. Don’t brown nose or act like you know more than you do, it irritates PDs and attendings. Be willing to learn from anyone.

Detailed Info by year:
  • Premed students:
No offense meant, but you truly have no idea yet. Knowing undergard or grad level biochem and cell bio is a tiny fraction of med school. This is likely the single biggest decision of your life which will impact you for the next 30+ years. Listen to a wide array of people who have been through it and plan accordingly. There is a spectrum of people in med school and more specifically at Carey ranging from completely miserable to reasonably content.

I don’t remember anyone doing backflips down the hall in OMS1 or 2.

OMS3 is pretty good

4th year is awesome!

Getting in: Take all the interviews you can until you get an acceptance (or 2), then you can afford to be choosey. IMO I would NOT forego an acceptance ANYWHERE to do an MBS or the like. There is no point in adding a year of school, debt etc. to maybe have a shot at a place you may or may not like better. This process is a grind; take the bird in hand.

Getting into Carey: If you get an interview you have a great chance to be accepted unless you completely blow the interview which is very unlikely If you are waitlisted, hang in there, you still have a shot. In my first year they added people from the list until the second day of classes.

Carey vs other DO schools vs MD vs Caribbean (classic SDN debate):

In truth I hated the first two preclinical years at Carey, there are plenty of specifics as to why by other students in this thread. After two years of rotations / residency interviews meeting other DO, MD, and IMG grads, no one has told me their first two years of med school were fantastic. Many if not most of the things that infuriate students at Carey exist at the other DO schools as well.

I had several options. I chose Carey because of the students. Honestly, if I did not have the friends I made at Carey, med school would have been a lot harder. There are gunners at every school, but I think we have fewer than most.

RE: Experiences of My MD friends

Less frustration in the preclinical years

Better environment to succeed on the boards. That statement is not speculative, I have seen course materials and syllabi from friends, and it is better. MD programs have had more time to ferret out things that do not benefit the students and typically are set in established academic environments, whereas DO schools are almost universally newer and still figuring out what works. Also, keep in mind we spend 20%+ of our time on OMT material which never appears on USMLE, theoretically giving them less material to study in the same time frame.

Better opportunities to match in highly competitive fields-see above + agreements with academic hospital systems, more help from admin, advisors, match stats, connections, etc.

MD students generally have less hands-on experience in years 3-4 because there are multiple layers of students, interns, residents, fellows, etc. above them getting in on procedures and patient care.
I believe the relationships a student can make with a preceptor at Carey is one of the better selling points of the school. My LORs to ERAS were outstanding, and even better my preceptors discussed them with me prior to submission. Without question, they opened doors for me to residency interviews I may have not had if I only worked with residents and had cookie cutter fill in the blank LORs.

The IMG grads (Caribbean or elsewhere) I have met have a rough go all around. DON’T DO IT UNLESS YOU HAVE NO OTHER OPTION!

They are basically on their own and self-study for the first two years and then struggle to get rotations in the U.S. I met an IMG MD at one of my interviews who was in her third application cycle and working as a nurse to pay loans until she found a residency. Access to good preceptors is extremely important along with a chance at an AI (acting internship) to get the attention of a specific program. The students in the Caribbean are encouraged to apply to 150-250 programs to try and get a residency spot. Again, not speculation but information offered by a Ross grad during an interview dinner. The IMGs I know lumped U.S. grads together, MD and DO when they compared their experience to ours.

  • OMS1

Goal: make it unscathed to OMS2.

Don’t get caught up in the med school frenzy, politics or the like. There is plenty of time for you to worry about residency, boards etc. Grind, study your power points, do as well as you can in class to lay a framework of knowledge onto which you will add to over the next 2 years. There are many things which will infuriate you but don’t let it get to you. Getting angry with things admin does will not help you nor will it change their outlook or position. It’s going to bring you down and your relationships with your studies, family, SOs, and friends will suffer if you’re irritated from things beyond your control. You are here; suck it up and remember you will soon be a doctor. Things will get better, I promise.

Enjoy your summer, maybe do some research if you can.

  • OMS2
Goal: don’t get tripped up by a class and work hard for Step/Level 1

You know how the classes work by now. Mind your attendance so you don’t fail but do what is best for your board studies. If you don’t learn from them in class, duck out and make better use of your time (while still maintaining 80%). Use your most precious commodity (time) wisely. Don’t sit back in class playing games or watching Netflix and then complain about barely passing or poor board scores. You will regret it at match time. Keep grinding and do things outside of school when you can to preserve your sanity.

UFAP (uworld first aid pathoma) or something similar. Pick a boards program and stick with it. Everybody and their brother is going to try and sell you something this year, emails, classmates trying to rep a program for a discount, sales people coming to school, you will see it all. Its all expensive. Everyone learns in their own way, looking back I used too many resources and spread myself too thin among them. I probably could have squeezed out more points by sticking with the tried and true resources. Keep in mind to do COMLEX style questions as well. It is a terribly written, vague exam but you MUST pass it.

Make sure you pass the COMSAE!!! It will mess up your study schedule if you don’t. They will probably force you to take some type of school sponsored course which is unlikely to help you as an individual. Be strategic about when you are taking step/level 1.

Begin to think about a field of interest but don’t fixate because it may change once you get into rotations.

  • OMS3
Goal: Shine on rotations and get those LORs, and work hard for step 2

There are many great resources for you, onlinemeded is fantastic, FA

Figure out what you want to do when you grow up and explore that field with your electives.

MAKE CONNECTIONS where you can, it’s going to get real, and very soon. Get those VSAS apps out ASAP. The AI spots fill up fast (don’t call it an “audition rotation”, It makes you sound less informed on how the system works). ACGME calls it acting internship (AI) or externship or sub-intership (Sub-I). There are also many good non-VSAS programs including FGH in Hattiesburg. In either case book the AI early.

Boards

Remember what worked or didn’t work for you on step/level 1. No reason to reinvent the wheel. And yes, COMLEX 2 is also terribly written.

Remember you must come back for OMT or OSCE which will take a big chunk away from step/level 2 studies. Plan and schedule accordingly.

The PE: its easy. Don’t stress it. If you have done well on your rotations, it will be fine. Learn how to craft a good SOAP note, you are going to need it for many years to come. Probably the best thing Carey does is prep you for the PE. Its shockingly similar. A+ for them on this one.

  • OMS4
Get AIs and WORK HARD!!!!!!!!!!!!!!!!!!!!!!!!!!! I cannot stress this enough. If you are borderline on paper but kill the rotation. it may be enough to get you that residency spot. If you are great on paper but flub the rotation, you are done. Anecdotally, I have seen it work both ways.

Be fully aware of what you will need for your applications, ex. SLOEs for EM are due by September 15th and plan ahead.

Get your application into ERAS on time!!!The Dean’s letter (or MPSE) comes out in October. Don’t wait for it. Most of your interviews will be booked long before it comes out if you are on your game.

Recognize your strengths and weaknesses and be realistic about your chances; seriously, you need to do this. Are you mediocre or not so great on paper? Do you get under people’s skin? Are your preceptor reviews less than stellar?

Most people who matched in my class did this. Many people who did not match were often unrealistic with their chances into their applied field or program, conversely, there were a few in our class who were tailor made for their field and applied appropriately and unfortunately, it didn’t work out for them. That’s not to say don’t shoot for the moon but have a plan B just in case. There are alternative paths if you do not match ex. TRI.

Remember that when you get to this point it is a job. Refer to the program director survey on the ACGME website to see what is important in obtaining a residency.

Among ALL specialties factors and importance for ranking applicants post interview # 1-4 have nothing to do with scores/ class rank (link below). I am by no means diminishing the importance of board scores. They are very important, and extremely important for a highly competitive specialty. But, keep in mind PD (program directors), faculty and residents want to know if they can work with you for long hours under high stress for 3-5 years straight. Being a decent and modest student/ person and having genuine interest in the field will serve you well in your journey.

http://www.nrmp.org/wp-content/uploads/2016/09/NRMP-2016-Program-Director-Survey.pdf

Good luck to all!

I'll be attending WCUCOM next year, and I have say this was very informative and pleasant to read. Thank you for taking the time to share your personal experiences with this program.
 
  • Like
Reactions: 1 users
Hi All,

I graduated Carey this past year and matched with my first choice residency program. I figured I would check in because it’s very rare when graduates post on this thread. Because I no longer have any ties to the program I feel I can speak very openly and freely about my experience.

I am doing this because I wish someone told me this information before I applied to med school and while I was in med school and before I applied to residency. I really hope it helps.

PART ONE of this post is new material since I have graduated which I think would be most applicable to current medical students as they approach the residency applications and beyond.


PART TWO is material I’m rebooting verbatim from a post I made while I was a fourth-year student that I hope will help both getting in and navigating your time in medical school. Caveat part two I’m no longer current student and have no idea what goes on campus today. I can only speak from my historical experience.

Ok here we go, if you’re just applying to med school you may want to scroll to part two...

PART ONE:
As I said above I matched my first choice program. I’m going to add this year I am part of my residency’s selection committee so I’ve been fortunate to see the process from the other side; see the interview applicants, read their personal statements, read their CVs, have input into who gets an interview etc.

My class has a whole, however, had a very poor match rate. At one point around match last year I knew the exact specifics but as they are no longer relevant to me I will speak anecdotally from memory while trying to remain as accurate as possible. My goal with this isn’t to argue statistics but merely to paint an overall picture of what the stats mean.

The numbers are +/- a few as I recall but they’re close enough for this conversation.

We matched approximately 56% of the graduating class (~90, however we began with 116 and gained several more along the way). That included both the AOA and ACGME match in total. In other words and again this is approximate 50/90 graduates actually matched.

Running total 50/90 matched

~18 people were successful in the AOA scramble. To be very clear on what that means these people did not match and had to personally call programs to ask if they had open positions they could fill after the match occurred.

2-3 successfully soap ACGME

Running total ~70/90 matched or scrambled/soap which means they are in the residency program without having to reapply.

~15 people were accepted into TRI (traditional rotating internship) programs. Think of this like the residency version of an MBS program. This means they did not match, they did not successfully scramble AOA or soap ACGME and they have added a year to the residency. TRI does not count as your basic three years of residency. A person in a TRI has to reapply to residency the following year while they are doing their internship. Statistically the odds of matching the second time around are very low. TRI interns are often stigmatized.

Running totals: approximately 85/90 found a home. Remember the people who landed a TRI will have to reapply for residency this year.

The rest will either reapply for residency this year or find job doing something else.

That said, many of my friends received their first choice program or a program somewhere at the top of their list because they were well prepared, well informed and realistic.

These numbers are very poor. It’s very evident there is a disconnect between what some of our graduates wanted and reality. There are a lot of ways to help your application and there a lot of ways to sink your application during this process.

There’s a common theme with many applicants who do not match in that is having unrealistic expectations. Often people apply the programs due to perceived status of a program or field.

When you’re entering this process take a very good look at yourself, your scores, your grades, your school, your preceptor evaluations, related research, perceived interest in the field, how well you are liked by your peers and preceptors and how well you interact with others in general. I promise you every one of these factors will come into play during your applications.

Factor all these together and look at the field and the specific programs to which you are applying. Are you shooting for dermatology or orthopedics which are historically extremely selective residency programs? If you’re board (USMLE) scores are in the 95th percentile along with your grades, you have multiple relevant publications, know someone in the field, and all your preceptors and classmates want to take you out and have a beer with you, then you probably have a pretty good shot. If you’re not checking those boxes, the reality is look for less competitive field or you may be applying again next year.

Don’t apply to a field based on perceived prestige or solely based on how much money you think you will make. I know many attending‘s in pediatrics, family medicine, internal medicine who make every bit as much as an orthopedist because they’re intelligent, use their time wisely and know how to maximize their worth.

Apply to a field because you love the work, because you’re passionate about it, and because you see yourself doing it 30 years from now, otherwise you will be 10 years into your career and hate what you do. The money will work itself out.

Basic tips:

Apply early

Show interest in the field

Be very specific to that field in your personal statement

Have your letters of recommendations specifically talk about that field. Don’t be stupid enough you have your LOR say “he will make a great surgeon” when you’re applying to internal medicine Even if you’re applying to surgery as well.

Don’t wait to take step 2. Plan ahead and get it done early.

Be gracious

Keep in mind your interview begins the moment you set foot in that town until the moment you leave. Every single person you encounter may have an impact on you getting accepted.

Secretaries and program coordinators have huge input and a direct line to the program directors. Irritate them and you have no chance; impressed them and they will speak highly of you.

I can’t believe I have to say this but I’ve seen it happen, don’t be a jerk, don’t brag about yourself and be genuinely interested.

Keep your personal statement to one page. get to the point and keep the readers attention. Most importantly it should relate to why you connect with the field to which you are applying.

Do something meaningful for your CV. Do something of meaning in your community or publish something or at least put something on there that’s interesting (run a marathon, climb a mountain, bungee jump over the moon whatever just make it memorable) No one cares that you were third vice chair in your med school club X. Don’t get me wrong, it’s very good to show specific interest in the club to the field you’re applying but don’t make that the centerpiece of your application.

Be thoughtful after your interview and send a thank you card or a thank you email to everyone who interviewed you, the program director, the program coordinator. Trust me it goes a long way.


PART TWO

There have been many repeat questions from new interviewees or newly accepted students. I am going to partially recycle a post I made a few months back with a lot of new updates and info. I have tried to make this post easy to scroll through its crazy long, I addedheadings by timeline year by year as follows:

Q&A – short and to the point
Premed - detailed
OMS1- detailed
OMS2- detailed
OMS3- detailed
OMS4 - detailed
I have been as unbiased as I can, based on my experience, successes and mistakes. I truly wish you all the best.

Context: This is my SDN swansong. I am a fourth year and recently matched my first-choice program. It felt REALLY GOOD to see DO next to my name in the residency contract! Here is my experience and perspective.

  • Q&A
Premed/OMS1 Q&A
Carey vs MD schools?
MD 100% see premed below.

Carey vs Caribbean schools?
Carey 100% see premed below.

Carey vs other DO schools?
Too much arguing on this topic on this thread. ACGME doesn’t care; seriously they don’t care if you’re from Carey or Touro, PCOM, TCOM, ZCOM, QCOM or wherever, unless the program is located by the specific DO school and they interact with the students on rotation. No ACGME program where I interviewed knew anything about Carey good or bad. They lumped all DO schools into the same general category and don’t know enough about them to stratify schools. See last few paragraphs in premed below.

Is Carey geared towards primary care?
Absolutely, its in the mission statement.

Is the attendance policy excessive?
Yes, according to most, but its there. Work around it.

Can I do 4th year away rotations away or in Hattiesburg?
Yes, and yes, but you need to plan them yourself. See OMS4 above

Best part about Carey?
Students 100%, also the newer, younger faculty seems to be pretty invested in the students.

Worst part?
Red tape all 4 years.

Best (most relevant to intended goal) educational aspect of Carey?
COMLEX level 2 PE prep.

Worst?
USMLE 1 or 2 anything

What is OMS1 or 2 like?
Ask an OMS1 or 2, its changed dramatically since my class.

Can I become a great Doctor after graduating Carey?
100%

OMS2-4 Q&A

Is there OMT work in OMS3 and 4?
Yes, to the very last OMS4 rotation.

Do I have to perform OMT on Rotations?
No

Should I take the USMLE?
Yes, if you want anything in ACGME other than FM or IM.

Is there an MD bias?
Not that I ever saw. Competent is competent. This seems to be a premed construct.

ACGME or AOA?
If you are class of 2019, IMO 100% ACGME if you can, IMO.
2020, this is a problem that will not exist, but the competition will no doubt increase because many AOA spots are going by the wayside.

Will I get much support with my application to residency?
No. I believe it will improve in the next few years though.

Has attending Carey helped or hurt my match?
UGH. I, along with many are very happy with match, but many are not. See OMS4 paragraphs below.

What about uber competitive specialties ex. ENT, derm etc.?
Unlikely based on past stats. If you want Derm, etc. go to a top 40 NIH funded MD school, kill the boards AND have a family member or close friend in that field. They are crazy hard to get no matter where you go!
Based on this year, if you apply from here 95%+ of you will get FM, IM, Peds, Psych, OB, ER, Gen Surg, Rads, Gas in that or similar order. There were a couple outliers this year, but this is what you can expect.

Will it help my match chances to apply in my home state?
Yes 100%

What is the 2018 placement / match rate this year?
Placement rate and match rate are vastly different things. Placement also includes AOA scramble, ACGME SOAP and post SOAP AOA empty spots, TRIs etc.
Placement rate TBD.
AOA match by school link
2018 Summary by College

Those numbers are convoluted, what does the above link mean?
Yep, that’s AOA. Basically, WCU matched similarly to the other DO schools in the AOA match.

What was the 2018 ACGME match rate?
Anecdotal at this point, nothing released by the school yet.

NRMP match link
http://www.nrmp.org/wp-content/uploads/2018/02/Results-and-Data-SMS-2018.pdf

What can I do to have the best match from Carey?
Work hard, make connections, do well on boards, be realistic about your field based on stats, be really cool to your attendings, residents, and other med students on rotations and interviews. Don’t brown nose or act like you know more than you do, it irritates PDs and attendings. Be willing to learn from anyone.

Detailed Info by year:
  • Premed students:
No offense meant, but you truly have no idea yet. Knowing undergard or grad level biochem and cell bio is a tiny fraction of med school. This is likely the single biggest decision of your life which will impact you for the next 30+ years. Listen to a wide array of people who have been through it and plan accordingly. There is a spectrum of people in med school and more specifically at Carey ranging from completely miserable to reasonably content.

I don’t remember anyone doing backflips down the hall in OMS1 or 2.

OMS3 is pretty good

4th year is awesome!

Getting in: Take all the interviews you can until you get an acceptance (or 2), then you can afford to be choosey. IMO I would NOT forego an acceptance ANYWHERE to do an MBS or the like. There is no point in adding a year of school, debt etc. to maybe have a shot at a place you may or may not like better. This process is a grind; take the bird in hand.

Getting into Carey: If you get an interview you have a great chance to be accepted unless you completely blow the interview which is very unlikely If you are waitlisted, hang in there, you still have a shot. In my first year they added people from the list until the second day of classes.

Carey vs other DO schools vs MD vs Caribbean (classic SDN debate):

In truth I hated the first two preclinical years at Carey, there are plenty of specifics as to why by other students in this thread. After two years of rotations / residency interviews meeting other DO, MD, and IMG grads, no one has told me their first two years of med school were fantastic. Many if not most of the things that infuriate students at Carey exist at the other DO schools as well.

I had several options. I chose Carey because of the students. Honestly, if I did not have the friends I made at Carey, med school would have been a lot harder. There are gunners at every school, but I think we have fewer than most.

RE: Experiences of My MD friends

Less frustration in the preclinical years

Better environment to succeed on the boards. That statement is not speculative, I have seen course materials and syllabi from friends, and it is better. MD programs have had more time to ferret out things that do not benefit the students and typically are set in established academic environments, whereas DO schools are almost universally newer and still figuring out what works. Also, keep in mind we spend 20%+ of our time on OMT material which never appears on USMLE, theoretically giving them less material to study in the same time frame.

Better opportunities to match in highly competitive fields-see above + agreements with academic hospital systems, more help from admin, advisors, match stats, connections, etc.

MD students generally have less hands-on experience in years 3-4 because there are multiple layers of students, interns, residents, fellows, etc. above them getting in on procedures and patient care.
I believe the relationships a student can make with a preceptor at Carey is one of the better selling points of the school. My LORs to ERAS were outstanding, and even better my preceptors discussed them with me prior to submission. Without question, they opened doors for me to residency interviews I may have not had if I only worked with residents and had cookie cutter fill in the blank LORs.

The IMG grads (Caribbean or elsewhere) I have met have a rough go all around. DON’T DO IT UNLESS YOU HAVE NO OTHER OPTION!

They are basically on their own and self-study for the first two years and then struggle to get rotations in the U.S. I met an IMG MD at one of my interviews who was in her third application cycle and working as a nurse to pay loans until she found a residency. Access to good preceptors is extremely important along with a chance at an AI (acting internship) to get the attention of a specific program. The students in the Caribbean are encouraged to apply to 150-250 programs to try and get a residency spot. Again, not speculation but information offered by a Ross grad during an interview dinner. The IMGs I know lumped U.S. grads together, MD and DO when they compared their experience to ours.

  • OMS1

Goal: make it unscathed to OMS2.

Don’t get caught up in the med school frenzy, politics or the like. There is plenty of time for you to worry about residency, boards etc. Grind, study your power points, do as well as you can in class to lay a framework of knowledge onto which you will add to over the next 2 years. There are many things which will infuriate you but don’t let it get to you. Getting angry with things admin does will not help you nor will it change their outlook or position. It’s going to bring you down and your relationships with your studies, family, SOs, and friends will suffer if you’re irritated from things beyond your control. You are here; suck it up and remember you will soon be a doctor. Things will get better, I promise.

Enjoy your summer, maybe do some research if you can.

  • OMS2
Goal: don’t get tripped up by a class and work hard for Step/Level 1

You know how the classes work by now. Mind your attendance so you don’t fail but do what is best for your board studies. If you don’t learn from them in class, duck out and make better use of your time (while still maintaining 80%). Use your most precious commodity (time) wisely. Don’t sit back in class playing games or watching Netflix and then complain about barely passing or poor board scores. You will regret it at match time. Keep grinding and do things outside of school when you can to preserve your sanity.

UFAP (uworld first aid pathoma) or something similar. Pick a boards program and stick with it. Everybody and their brother is going to try and sell you something this year, emails, classmates trying to rep a program for a discount, sales people coming to school, you will see it all. Its all expensive. Everyone learns in their own way, looking back I used too many resources and spread myself too thin among them. I probably could have squeezed out more points by sticking with the tried and true resources. Keep in mind to do COMLEX style questions as well. It is a terribly written, vague exam but you MUST pass it.

Make sure you pass the COMSAE!!! It will mess up your study schedule if you don’t. They will probably force you to take some type of school sponsored course which is unlikely to help you as an individual. Be strategic about when you are taking step/level 1.

Begin to think about a field of interest but don’t fixate because it may change once you get into rotations.

  • OMS3
Goal: Shine on rotations and get those LORs, and work hard for step 2

There are many great resources for you, onlinemeded is fantastic, FA

Figure out what you want to do when you grow up and explore that field with your electives.

MAKE CONNECTIONS where you can, it’s going to get real, and very soon. Get those VSAS apps out ASAP. The AI spots fill up fast (don’t call it an “audition rotation”, It makes you sound less informed on how the system works). ACGME calls it acting internship (AI) or externship or sub-intership (Sub-I). There are also many good non-VSAS programs including FGH in Hattiesburg. In either case book the AI early.

Boards

Remember what worked or didn’t work for you on step/level 1. No reason to reinvent the wheel. And yes, COMLEX 2 is also terribly written.

Remember you must come back for OMT or OSCE which will take a big chunk away from step/level 2 studies. Plan and schedule accordingly.

The PE: its easy. Don’t stress it. If you have done well on your rotations, it will be fine. Learn how to craft a good SOAP note, you are going to need it for many years to come. Probably the best thing Carey does is prep you for the PE. Its shockingly similar. A+ for them on this one.

  • OMS4
Get AIs and WORK HARD!!!!!!!!!!!!!!!!!!!!!!!!!!! I cannot stress this enough. If you are borderline on paper but kill the rotation. it may be enough to get you that residency spot. If you are great on paper but flub the rotation, you are done. Anecdotally, I have seen it work both ways.

Be fully aware of what you will need for your applications, ex. SLOEs for EM are due by September 15th and plan ahead.

Get your application into ERAS on time!!!The Dean’s letter (or MPSE) comes out in October. Don’t wait for it. Most of your interviews will be booked long before it comes out if you are on your game.

Recognize your strengths and weaknesses and be realistic about your chances; seriously, you need to do this. Are you mediocre or not so great on paper? Do you get under people’s skin? Are your preceptor reviews less than stellar?

Most people who matched in my class did this. Many people who did not match were often unrealistic with their chances into their applied field or program, conversely, there were a few in our class who were tailor made for their field and applied appropriately and unfortunately, it didn’t work out for them. That’s not to say don’t shoot for the moon but have a plan B just in case. There are alternative paths if you do not match ex. TRI.

Remember that when you get to this point it is a job. Refer to the program director survey on the ACGME website to see what is important in obtaining a residency.

Among ALL specialties factors and importance for ranking applicants post interview # 1-4 have nothing to do with scores/ class rank (link below). I am by no means diminishing the importance of board scores. They are very important, and extremely important for a highly competitive specialty. But, keep in mind PD (program directors), faculty and residents want to know if they can work with you for long hours under high stress for 3-5 years straight. Being a decent and modest student/ person and having genuine interest in the field will serve you well in your journey.

http://www.nrmp.org/wp-content/uploads/2016/09/NRMP-2016-Program-Director-Survey.pdf

Good luck to all!

Incredibly useful post, and I'm a current student.
I really appreciate you dropping by with your knowledge, and best of luck in residency!
 
  • Like
Reactions: 1 user
Our dedicated starts Mid-May, most upperclassmen took their exams throughout July and some into August, which would be about 7-11 weeks.
 
  • Like
Reactions: 1 users
How long are 2nd years given for board prep?
We are given an equivalent of 0 hours and 0 minutes and 0 seconds for board prep during first 2 years while school is in session. What you give yourself is how much time you will have for board prep.

You will be surrounded by people who are also in the same boat so you can be stressed out and panicking about the boards together, while still not being able to study for boards. It is an exciting time.
 
Last edited:
We are given an equivalent of 0 hours and 0 minutes and 0 seconds for board prep during first 2 years while school is in session. What you give yourself is how much time you will have for board prep.

You will be surrounded by people who are also in the same boat so you can be stressed out and panicking about the boards together, while still not being able to study for boards. It is an exciting time.

I may be wrong, but I believe RockyTopDoc94 may have been asking about the 'Dedicated Period' at the end of 2nd year before 3rd year starts.
 
  • Like
Reactions: 1 user
How long are 2nd years given for board prep?

We are given an equivalent of 0 hours and 0 minutes and 0 seconds for board prep during first 2 years while school is in session. What you give yourself is how much time you will have for board prep.

You will be surrounded by people who are also in the same boat so you can be stressed out and panicking about the boards together, while still not being able to study for boards. It is an exciting time.

2nd year students are given 1 month in December (Christmas break) and then another 8-10 weeks for "dedicated study" depending on when you decide you want to take your boards.
 
  • Like
Reactions: 1 user
It's weird that theyre already interviewing for the waitlist so early in the cycle.
 
Does WCU do silent rejections?
Its been a few months and I still haven't even received a secondary.
 
Top